Fermat's Theorem won't help you. Or will it?

Let xx, yy be coprime positive integers with xy>1xy > 1, and let nn be an even positive integer. Prove that xn+ynx^n + y^n is not divisible by x+yx + y.

#NumberTheory #Fermat'sLastTheorem #Finn #Proof #CanYouProveIt?

Note by Finn Hulse
7 years, 1 month ago

No vote yet
1 vote

  Easy Math Editor

This discussion board is a place to discuss our Daily Challenges and the math and science related to those challenges. Explanations are more than just a solution — they should explain the steps and thinking strategies that you used to obtain the solution. Comments should further the discussion of math and science.

When posting on Brilliant:

  • Use the emojis to react to an explanation, whether you're congratulating a job well done , or just really confused .
  • Ask specific questions about the challenge or the steps in somebody's explanation. Well-posed questions can add a lot to the discussion, but posting "I don't understand!" doesn't help anyone.
  • Try to contribute something new to the discussion, whether it is an extension, generalization or other idea related to the challenge.
  • Stay on topic — we're all here to learn more about math and science, not to hear about your favorite get-rich-quick scheme or current world events.

MarkdownAppears as
*italics* or _italics_ italics
**bold** or __bold__ bold

- bulleted
- list

  • bulleted
  • list

1. numbered
2. list

  1. numbered
  2. list
Note: you must add a full line of space before and after lists for them to show up correctly
paragraph 1

paragraph 2

paragraph 1

paragraph 2

[example link](https://brilliant.org)example link
> This is a quote
This is a quote
    # I indented these lines
    # 4 spaces, and now they show
    # up as a code block.

    print "hello world"
# I indented these lines
# 4 spaces, and now they show
# up as a code block.

print "hello world"
MathAppears as
Remember to wrap math in \( ... \) or \[ ... \] to ensure proper formatting.
2 \times 3 2×3 2 \times 3
2^{34} 234 2^{34}
a_{i-1} ai1 a_{i-1}
\frac{2}{3} 23 \frac{2}{3}
\sqrt{2} 2 \sqrt{2}
\sum_{i=1}^3 i=13 \sum_{i=1}^3
\sin \theta sinθ \sin \theta
\boxed{123} 123 \boxed{123}

Comments

Let x+y=tx+y=t
Since xymodtx \equiv -y \mod t, we have,
xn+yn2ynmodtx^n+y^n \equiv 2y^n \mod t
If the above modulo equals 00, then t2ynt|2y^n which is clearly impossible since tt does not divide 22 (xy>1    t3xy>1 \implies t \ge 3) and tt does not divide yny^n (xx and yy are coprime).

Ishan Singh - 7 years, 1 month ago

Log in to reply

Looks good, but you need to be slightly careful, especially if you haven't used all the conditions in the problem. (Given that it's an olympiad problem, and they tend to be provide just enough details.) If so, think about it. Is that a necessary condition? If yes, then you need to use it. If not, then you might want to explain why the condition isn't needed.

In this case, the statement is true for (x,y)=(1,1) (x,y) = (1,1) , which contradicts your claim that we can solutions "only if y=0 y = 0 ".

Calvin Lin Staff - 7 years, 1 month ago

Log in to reply

@Calvin Lin I guess this should be suffice.

Ishan Singh - 6 years, 3 months ago

Why would you need Fermat's Last Theorem? What is wrong with a 'remainder-factor-theorem' argument like @Ishan Singh 's? Am I missing something obvious?

Mursalin Habib - 7 years, 1 month ago

Log in to reply

I added that just to make you think a little bit. :D

Finn Hulse - 7 years, 1 month ago

I am pretty sure this is correct.

For any xn+ynx^n + y^n, where xx and yy are coprime positive integers and nn is an even positive integer, the equation is irreducible, i.e. it can not be factorised. Hence it has no integer root. But since the question states that xx and yy are integers and the sum of two integers is always an integer, xn+ynx^n + y^n is not divisible by x+yx + y.

Sharky Kesa - 7 years, 1 month ago

Log in to reply

Do not confuse a statement about divisibility of polynomials, with a statement about divisibility of integers.

Calvin Lin Staff - 7 years, 1 month ago

Log in to reply

Yeah.

Finn Hulse - 7 years, 1 month ago
×

Problem Loading...

Note Loading...

Set Loading...